Which descriptions can describe more than one triangle? Select two options.
I side lengths of 6 ft, 8 ft, and 10 ft
D
angle measurements of 35°, 35°, and 110°
EI
angle measurements of 30°, 40°, and 50°
I angle measurements of 40°, 60°, and 80°
side lengths of 4 cm, 6 cm, and 9 cm 6

Answers

Answer 1

Answer: (35,35,110) and (40,60,80)

Step-by-step explanation:

If given all 3 side lengths, it can only describe one triangle. Angles alone are not enough to guarantee how a triangle looks.

side lengths of 6 ft, 8 ft, and 10 ft - no because all sides are defined

angle measurements of 35°, 35°, and 110° - yes because three angles can describe more than one triangle

angle measurements of 30°, 40°, and 50° - no because the sum of the angles is 120 degrees. Triangles have a combined total of 180 degrees

angle measurements of 40°, 60°, and 80° - yes because 3 angles

side lengths of 4 cm, 6 cm, and 9 cm 6 - no because all sides are defined

Brainliest please :D


Related Questions

One side of a triangle is 10 feet long, the second side is 20 feet long. What is the
range of possible values for the third side of the triangle? Why?

Answers

The range for the length of the third side is it must be greater than 10 feet and less than 30 feet.

What is the condition for the sides of triangles?

The triangle inequality states that the sum of the lengths of any two sides of a triangle must be greater than or equal to the length of the third side. That sum can equal the length of the third side only in the case of a degenerate triangle, one with collinear vertices.

The length of the third side had to be greater than the difference between the two given sides and less than the sum of the two given sides.

(20 – 10) < S <(20+ 10) ; 10< S < 30

Therefore, the range for the length of the third side is it must be greater than 10 feet and less than 30 feet.

To learn more about the triangles visit:

https://brainly.com/question/2773823.

#SPJ1

1. Annie and her friends are playing a game called Doubles. In the game, a player
rolls two number cubes at the same time. The outcome of each roll is the sum of the
two number cubes. Extra points are scored for rolling doubles the same number
on both number cubes.
-
Part A: Complete this table to show the sample space for each roll in the game. (6
points)

HELP ASAP WILL GIVE BRAINLIEST!!!!

Answers

The sample space is a combination of these two dice as we have shown in the attachment

Die A = {1,2,3,4,5,6}Die b = {1,2,3,4,5,6}

What is sample space?

This is the defined to be the set of all of the possible outcomes that can exist or be possible in a random experiment.

The question has two dice. These dice are made up of numbers from 1 to 6  each.

Die A = {1,2,3,4,5,6}

Die b = {1,2,3,4,5,6}

The sample space would be pairs of two of these two dices. The total number in the sample space would be 36.

Read more on sample space here:

https://brainly.com/question/10558496

#SPJ1

The area of a rectangular pool is (x 2 + 17x + 72) square meters. There is a 3-meter-wide concrete
walkway around the pool. Write expressions to represent the dimensions of the outside border of the
walkway. (Lesson 8.1)

Answers

Step-by-step explanation:

the area is

(x² + 17x + 72) m²

it is the result of the usual

length × width

calculation.

length = x + g

width = x + h

length × width = x² + gx + hx + ab = x² + (g+h)x + gh =

= x² + 17x + 72

g + h = 17

gh = 72

just by looking at it that gives us 9 and 8 as g and h.

FYI - formally to calculate them we can say

g = 17 - h

(17 - h)h = 72

17h - h² = 72

h² - 17h + 72 = 0

the solution to such a quadratic equation is

x = (-b ± sqrt(b² - 4ac))/(2a)

in our case

x = h

a = 1

b = -17

c = 72

h = (17 ± sqrt(17² - 4×1×72))/(2×1) =

= (17 ± sqrt(289 - 288))/2 = (17 ± 1)/2

h1 = (17 + 1)/2 = 18/2 = 9

h2 = (17 - 1)/2 = 16/2 = 8

as we can see

g = 17 - h

if we say h = 9, then g = 8.

if we say h = 8, then g = 9.

so, these are our solutions.

we simply pick that length is longer than width, so, g = 9, h = 8.

the area including the walkway is then

(length + 3)(width + 3) =

= (x + 9 + 3)(x + 8 + 3) = (x + 12)(x + 11) m²

and the dimensions incl. the walkway are

length = x + 9 + 3 = x + 12

width = x + 8 + 3 = x + 11

Select the correct answer.
Select the simplification that accurately explains the following statement.
3/7 = 73
O A.
100
B.
3
(71)² = 73.71.7371-1-1 = 78 = 7² = 7
(71)² = 78.73.7} = 7.73 = 3-1-7=1-7=7
3
C.
(7³)³ = 7³. 7³. 73 = 73 + } + ³ = 7³ = 7¹ = 7
D. (73)³ = 7³. 73. 73 = 7 · (++) = 7 ⋅ = 7 · 1 = 7

Answers

The simplification that represents ∛7=[tex]7^{1/3}[/tex] is [tex](7^{1/3} )^{3} =7^{1/3} .7^{1/3} .7^{1/3} =7^{1/3+1/3+1/3} =7^{1}=7[/tex]

How to simplify an expression?

The simplification that accurately explains the statement ∛7 = [tex]7^{1/3}[/tex] is as follows;

Therefore, the simplification is as follows

[tex](7^{1/3} )^{3} =7^{1/3} .7^{1/3} .7^{1/3} =7^{1/3+1/3+1/3} =7^{1}=7[/tex]

learn more simplification here: https://brainly.com/question/17350733

#SPJ1

Help me to solve this question​

Answers

Answer:

[tex]\left \{ {{\theta_1=\frac{\pi }{4} } \atop {\theta_2=\frac{3\pi }{4} }} \right. .[/tex]

Step-by-step explanation:

[tex]2*cos^2\theta-1=0\ \ \ \ (0^0\leq \theta\leq 360^0)\ \ \ \ \ \theta=?\\2*cos^2\theta-(sin^2\theta+cos^2\theta)=0\\2*cos^2\theta-sin^2\theta-cos^2\theta=0\\cos^2\theta-sin^2\theta=0\\cos2\theta=0\\0^0\leq \theta\leq 360^0\ \ \ \ \Rightarrow\\\left \{ {{2\theta=\frac{\pi }{2}\ |:2 } \atop {2\theta=\frac{3\pi }{2}\ |:2 }} \right. \ \ \ \ \left \{ {{\theta_1=\frac{\pi }{4} } \atop {\theta_2=\frac{3\pi }{4} }} \right. .[/tex]

Good luck and have a nice day!

a triangle with base 14, sides 12 and 6, and height 5. what is the area of the triangle? a = one-half(14)(6) = 42 square units a = one-half(12)(5) = 30 square units a = one-half(14)(5) = 35 square units a = one-half(6)(5) = 15 square units

Answers

The triangle with base 14, sides 12 and 6, will have an area, a = (1/2)(14)(5) = 35 square units. Hence, the 3rd option is the correct choice.

The area of a plane figure is the space occupied within its boundary.

The area of a triangle is given as,

a = (1/2)(b)(h) sq. units, where a is the area of the triangle, b is its base, and h is its height.

In the question, we have been given a triangle with a base of 14, sides 12 and 6, and a height of 5.

We are asked to determine the area of this triangle.

To determine the area of the triangle, we will use the formula:

a = (1/2)(b)(h) sq. units, where a is the area of the triangle, b is its base, and h is its height.

Now, we are given that the base is 14 and the height is 5, thus substituting b = 14, and h = 5, in the above formula, we get:

a = (1/2)(14)(5) sq. units.

or, a = 35 sq. units.

Comparing this with the given options, we can choose the third option.

Thus, the triangle with base 14, sides 12 and 6, will have an area, a = (1/2)(14)(5) = 35 square units. Hence, the 3rd option is the correct choice.

Learn more about the area of a triangle at

https://brainly.com/question/17335144

#SPJ4

Susan's calculator has a key that replaces the number displayed with its cube. If a² is displayed, how many times must Susan press the " cubing" key to display a number that is greater than 10^9?

Please explain it step-by-step! Also, I will give you a brainlist!!

Answers

If we want to replace a input and display an output greater than 10⁹ in Susan's calculator, then we want at least 3 consecutive operations.

How to determine a power by a "cubing" key

In accordance with the statement, Susan's calculator performs the following operation by pressing the "cubing" key:

x → x³

If we need to display a number that is greater than 10⁹, then we need to press the "cubing" key as many as needed. By algebra we have the following property for a power:

[tex](x^{m})^{n} = x^{m\cdot n}[/tex]     (1)

Since we have a "cubing" key and we are supposed to make consecutive operation, we derive the following operation:

x → [tex]x^{3^{i}}[/tex], where i is the number of consecutive "cubing" operations.

If we want to replace a input and display an output greater than 10⁹ in Susan's calculator, then we want at least 3 consecutive operations.

Remark

The statement presents typing mistakes, correct form is shown below:

Susan's calculator has a key that replaces the number displayed with its cube. If a³, is displayed, how many time must Susan press the "cubing" key to display a number that is greater than 10⁹.

To learn more on real numbers: https://brainly.com/question/551408

#SPJ1

Write the interval using interval notation to describe the set of values shown above.
Use "U" between the two intervals. Use "oo" (two lower case o's) for
∞.

Answers

The interval on the number line is (-∞, 2) U [3, ∞)

How to determine the interval?

From the number line, we have the following inequalities

x < 2 and x ≥ 3

In inequalities, < and > are represented by brackets i.e. ( and )

While ≤ and ≥ are represented by square brackets i.e. [ and ]

So, we have:

x < 2 and x ≥ 3 = 2) and [3

The intervals must be closed and opened with infinity symbols.

So, we have:

x < 2 and x ≥ 3 = (-∞, 2) and [3, ∞)

Replace and with U

x < 2 and x ≥ 3 = (-∞, 2) U [3, ∞)

Hence, the interval on the number line is (-∞, 2) U [3, ∞)

Read more about interval notation at:

https://brainly.com/question/13048073

#SPJ1

For the following exercises, determine whether the relation is a function.
1. {(a, b), (c, d), (e, d)}

Answers

Answer:

The given relation [tex]$\{(a, b),(c, d),(e, d)\}$[/tex] is a function.

Step-by-step explanation:

A relation [tex]$\{(a, b),(c, d),(e, d)\}$[/tex] is given.

It is required to determine whether the given relation is a function.

To determine whether the given function is a relation, identify the domain and range and then check whether the given relation is a function.

Step 1 of 1

The given relation is {(a, b),(c, d),(e, d)}.

The set of the first components of each ordered pair is called the domain.

From the relation, the domain is {a, c, e}.

The set of the second components of each ordered the range.

From the relation, the range is {b, d, d}.

The given relation is a function. But it is not a one-to-one function.

mrs chapagain borrowed rs 9900 from a bank at the rate of 8% per annum. how much amount should she pay at the end of 2 years​

Answers

Answer:

If compounding = rs 11547.36

If not compounding = rs 11484

Find the area in square centimeters of the composite, shape shown
below. Enter only a number as your answer.
A
11 cm
B
E
13 cm
18 cm
D
7 cm
C

Answers

The area of the composite shape is 136 square cm

Area of a composite function

The area of the composite function is expressed as;

Area = area of triangle + area of rectangle

Area of the composite shape = (1/2* 4 * 5) + (7*18)

Area of the composite shape = 10 + 126

Area of the composite shape = 136 square cm

Hence the area of the composite shape is 136 square cm

Learn more on area of composite shape here: https://brainly.com/question/8370446

#SPJ1

ABC is a right angled triangle. If b=90 ac=96 c=30. What is ab?

Answers

Answer:

ab=48

Step-by-step explanation:

96/2=48

On a coordinate plane, quadrilateral J K L M with diagonals is shown. Point J is at (4, 5), point K is at (5, 1), point L is at (1, 2), and point M is at (1, 5).
Which statement proves that quadrilateral JKLM is a kite?

∠M is a right angle and MK bisects ∠LMJ.
LM = JM = 3 and JK = LK = StartRoot 17 EndRoot.
MK intersects LJ at its midpoint.
The slope of MK is –1 and the slope of LJ is 1.

Answers

The statement that proves that that quadrilateral JKLM is a kite is: (Option B) which indicates that:
LM = MJ = 3; a

JK = KL = √17

What is the explanation for the above?

We know the following:

J = (4,5)

K = (5,1)

L = (1,2)

M = (1, 5)

What is the proof?

We must commence by computing the length of the sides of the quadrilateral using:

D = [tex]\sqrt{((x1-x2)^{2} + (y1 -y2)^{2} }[/tex]

JK = [tex]\sqrt{(4-5)^{2} + (5-1)^{2} }[/tex]

= [tex]\sqrt{17}[/tex]

KL = [tex]\sqrt{(5-1)^{2} + (1-2)^{2} }[/tex]

= [tex]\sqrt{17}[/tex]

LM = [tex]\sqrt{(1-1)^{2} + (5-5)^{2} }[/tex]

= [tex]\sqrt{9}[/tex]

= 3

MJ = [tex]\sqrt{(1-4)^{2} + (5-5)^{2} }[/tex]

= [tex]\sqrt{9}[/tex]

= 3

From the above, it is clear that

LM = MJ = 3; and

JK = KL = [tex]\sqrt{17}[/tex]

QED

Learn more about Proof and Kites at;
https://brainly.com/question/12160818
#SPJ1

Answer: B

Step-by-step explanation:

Enter the correct answer in the box.
What is the inverse of function f?

f(x)=-7x-4

Answers

Answer:

The inverse is  -1/7(x+4)

Step-by-step explanation:

We have the function

y = -7x-4

To find the inverse, exchange x and y

x = -7y -4

Solve for y

Add 4 to each side

x+4 = -7y -4+4

x+4 = -7y

Divide each side by -7

-1/7(x+4) = -7y/-7

-1/7(x+4) = y

The inverse is  -1/7(x+4)

What are the domain and range of the exponential function below?
F(x) = 4x + 3
A. Domain: All real numbers Range: All real numbers greater than 0
B. Domain: All real numbers greater than 0 Range: All real numbers greater than 3
C. Domain: All real numbers greater than 0 Range: All real numbers greater than 0
D. Domain: All real numbers Range: All real numbers greater than 3

Answers

The domain and the range of the function are all set of real numbers

How to determine the domain and the range?

The function is given as:

f(x) = 4x + 3

The above function is a linear function

Linear functions have a domain and a range of all set of real numbers

Hence, the domain and the range of the function are all set of real numbers

Read more about domain and range at:

https://brainly.com/question/1632425

#SPJ1

CAN SOMEONE HELP ME
Triangle not drawn to scale
Given: measure of angle A = 15 degrees, b = 16, and
c = 18. What is the length of a to the nearest tenth?

Answers

Answer:

a ≈ 4.9

Step-by-step explanation:

using the Cosine Rule in Δ ABC

a² = b² + c² - (2bc cosA )

   = 16² + 18² - (2 × 16 × 18 × cos15° )

   = 256 + 324 - (576cos15° )

   = 580 - 576cos15

   = 23.6267 ( take square root of both sides )

a = [tex]\sqrt{23.6267}[/tex]

   ≈ 4.9 ( to the nearest tenth )

The difference of two complementary angles is 48. Find the measures of the two angles.
The smaller angle is
Submit Question
and the larger angle is
B

Answers

The larger angle is 55 deg at the larger angle.

We have to know which angles are called Complementary Angles...

if we add two angles and the total will be equal to 90 deg

let's say one angle is x and  another angle is  y

as they are complementary angles from the definition we can write the equation x + y =  90

one angle is 20 deg more  than another one  so from the above condition  we write x = 20 + y

Substitute the value of x  in the first eaquation we get, 20 + y + y =  90

Then,  20 + 2y =  90

subtracting 20 from both sides,

 2y  =  90-20 = 70  

 dividing both sides by 2,

2y/ 2 = 70 / 2

 y  =  35

 So we got one angle is 35 deg

another angle is  35 + 20 = 55 deg.

Our answer is 55 deg the larger angle.

To learn more about the measure angle visit:

https://brainly.com/question/25716982

#SPJ1

Class A has 30 pupils and class B has 26 pupils.
Both classes sit the same maths test.
The mean score for class A is 70.
The mean score for both classes is 76.
What is the mean score (rounded to 2 DP) in the maths test for class B?

Answers

The mean score of class B is 82. 92

How to determine the mean score

Class A = 30 pupils

Class B = 26 pupils

Mean score for class A = 70

Mean score of class B = unknown

Total mean score = 76

We have, [tex]\frac{(76 *( 30 + 26) - 70 *30) }{26}[/tex]

Then simply,

Calculate the sum or difference between the classes = [tex]\frac{76 * 56 - 70 * 30}{26}[/tex]

Calculate the product or quotient of the classes = [tex]\frac{4256 - 70 * 30}{26}[/tex]

= [tex]\frac{4256 - 2100}{26}[/tex]

= [tex]\frac{2156}{26}[/tex]

= [tex]\frac{1078}{13}[/tex]

= [tex]82. 92[/tex]

Thus, the mean score of class B is 82. 92

Learn more about mean score here:

https://brainly.com/question/17203075

#SPJ1

104, 137, 154, 131, x; mean= 130

Answers

Hello!

The mean is the average of the terms.

⇒ Mean = Sum of terms / Number of terms

⇒ 130 = 104 + 137 + 154 + 131 + x / 5

⇒ x + 526 = 650

x = 124

∴ The missing number is 124.

GIVING BRAINLIEST TO THE BEST ANSWER! Choose the proportion to use to solve side LM.


A. 9/13.5 = x/11


B. 9/11 = 13.5/x


C. 9/13.5 = x/21


D. 9/11 = x/21

(the / represents fractions)

Answers

The equation that can be used to find the proportion to use to solve side LM is 9/13.5 = 14/LM

Similarity theorem of triangle

In order to find the measure of LM, we will take the ratio of the sides of the similar triangles.

Using the equation as shown

DF/EF = LK/LM

DF/LK = EF/LM

Substitute the measure of the sides

9/13.5 = 14/LM

Hence the equation that can be used to find the proportion to use to solve side LM is 9/13.5 = 14/LM

Learn more on similarity theorem here: https://brainly.com/question/4163594

#SPJ1

Determine whether you would use the Law of Sines or the Law of Cosines to solve each triangle. Do not solve the triangle.

Answers

A triangle is a three-edged polygon with three vertices. Only the law of Sine can be applied to solve the triangle.

What is a triangle?

A triangle is a three-edged polygon with three vertices. It is a fundamental form in geometry. The sum of all the angles of a triangle is always equal to 180°.

The sum of all three angles can be written as,

118° + 22° + x = 180°

x = 40°

For the sine rule to be applicable, two angles and a side or an angle and two sides are needed. Therefore, the law of Sine can be applied to solve the triangle.

For the cosine rule to be applicable, two sides and an angle between them is needed to be known, hence, the law of cosine can not be applied.

Hence, Only the law of Sine can be applied to solve the triangle.

Learn more about Triangle:

https://brainly.com/question/2773823

#SPJ1

Which graph shows y=3⌈x⌉+1?

Answers

The graph that shows y=3⌈x⌉ + 1 from the given options is the Third graph as shown attached.

Which graph shows the function best?

The third graph shows the function and this can be tested as follows:

When x = 2, y = 7:

y = 3x + 1

= 3 x 3 + 1

= 7

When x = 3, y = 10:

y = 3 x 3 +1

= 10

When x = 1, y = 4

y = 3 x 1 + 1

= 4

This shows that the line on the third graph is well represented by the function, y=3⌈x⌉+1.

Find out more on straight-line functions at https://brainly.com/question/22487519.

#SPJ1

I really need help asap! ty!

Answers

The length of AL in the triangle is 12cm.

How to calculate the value?

From the information given, angle C = 90° and BAC is given as 2ABC.

In this case, the length of angle BC is given as 24cm. Therefore, the length of AL will be:

= 24/2

= 12..

Therefore, AL = 12cm

Learn more about triangles on:

brainly.com/question/1058720

#SPJ1

How many real solutions does this system of equations have? x² + y² = 25 and x-3y=15​

Answers

[tex]x - 3y = 15 \\ x = 15 + 3y[/tex]

[tex](15 + 3y) {}^{2} + y {}^{2} = 25[/tex]

[tex]225 + 90y + 9y {}^{2} + y {}^{2} = 25[/tex]

[tex]10y {}^{2} + 90y + 200 = 0[/tex]

[tex]y {}^{2} + 9y + 20 = 0[/tex]

[tex](y + 4)(y + 5) = 0[/tex]

[tex]y = - 4 \: \: \: \: \: \: y = - 5[/tex]

[tex]for \: y= - 4 \\ x = 15 + 3( - 4) = 15 - 12 = 3[/tex]

[tex]for \: y = - 5 \\ x = 15 + 3( - 5) = 15 - 15 = 0[/tex]

For the following exercises, find the x- and y-intercepts of the graphs of each function.
27. f(x) = −3| x − 2 | − 1

Answers

For the function f(x) = -3|x - 2| - 1, we have that:

The function has no x-intercepts.The y-intercept is of y = -7.

What is the x-intercept of a function?

The x-intercept of a function is the value of x when y = f(x) = 0, hence:

0 = -3|x - 2| - 1

-3|x - 2| = 1

|x - 2| = -1/3

The absolute value cannot be negative, hence the function has no x-intercepts.

What is the y-intercept of a function?

The x-intercept of a function is the value of y = f(x) when x = 0, hence:

f(0) = -3|0 - 2| - 1 = -6 - 1 = -7.

The y-intercept is of y = -7.

More can be learned about intercepts at https://brainly.com/question/24737967

#SPJ1

Rationalize 5+2√3 / 7+4√3​

Answers

Answer:

Answer is in the 2nd attachment

Step-by-step explanation:

the first attachment is the work done.

The breaking strengths of cables produced by a certain manufacturer have historically had a mean of pounds and a standard deviation of pounds. The company believes that, due to an improvement in the manufacturing process, the mean breaking strength, , of the cables is now greater than pounds. To see if this is the case, newly manufactured cables are randomly chosen and tested, and their mean breaking strength is found to be pounds. Assume that the population is normally distributed. Can we support, at the level of significance, the claim that the population mean breaking strength of the newly-manufactured cables is greater than pounds

Answers

No, we cannot claim that the population mean breaking strength of the newly- manufactured cables is greater than pounds.

Given mean=1900, standard deviation=65, sample size=150, sample mean=1902, level of significance=0.01.

The hypothesis are:

[tex]H_{0}:\\[/tex]μ=1900

[tex]H_{1}:[/tex]μ>1900

We have to use z test as the sample size is large and we know the population standard deviation.

z=(x-μ)/σ/[tex]\sqrt{n}[/tex]

z=(1902-1900)/65/[tex]\sqrt{150}[/tex]

z=0.38

finding the p value:

p value=P(Z>z)

=P(Z>0.38)

=1-P(Z<0.38)

from the z table we get;

P(Z<0.38)=0.6480

Therefore p value=1-0.6480

=0.3520

If the p value is less than 0.01 then we reject the [tex]H_{0}[/tex] otherwise we will reject the null hypothesis.

In this problem we also reject the null hypothesis.

Hence  we cannot claim that the population mean breaking strength of the newly- manufactured cables is greater than pounds.

Learn more about z test at https://brainly.com/question/14453510

#SPJ4

Question is incomplete. It should includes:

 mean=1900,

standard deviation=65,

sample size=150,

sample mean=1902,

level of significance=0.01.

A property agent sells a house for $320 000, of which he receives a commission of 1.5 %.
Calculate the amount of commission the agent receives.

Answers

The answer for this is 320000 of the answer because I did the math


Pls help on algebra homework. Will mark brainliest!

Answers

Answer:

H)

Step-by-step explanation:

First, Length XY is 20 centimeters. Since YZ = 3/5 XY, [tex]\frac{3}5\times20=12[/tex]

YZ = 12 + 20 = 32 cm, hence H)

Hope you have a nice day! (Pls mark brainliest)

Write an equation for the graph below in terms of x

Answers

Answer: Y=-x-3
Explanation: Slope= rise/run = -1, the y-intercept is -3. Plug the numbers into the y=mx+b equation
Other Questions
Alexander Efland has a savings account that earns 5.5% interest compounded daily. On May 5, the amount in the account was $28,214.35. How much interest will the money earn in the next 90 days? Helpp plleeaaseee!!!!!! Which flexibility technique may be assisted by or utilizes a stretching tool like a towel or stretching strap? Dynamic stretching Myofascial release Pre-contraction stretching Ballistic stretching A survey revealed that 36% of people are entertained by reading books, 32% are entertained by watching TV, and 13% are entertained by both books and TV. What is the probability that a person will be entertained by either books or TV Ahora Ud. es Sonia. Digale a su nuevo amigo o amiga qu puede hacer en su casa, qu le gusta a Ud. y cmo son sus amigas de Costa Rica. Escriba y despus grabe 5 oraciones. (20 puntos) A survey was conducted among 1000 people. Of these 595 are Democrats, 595 wear glasses and 550 like ice-cream; 395 of them are Democrats who wear glasses, 350 of them are Democrats who like ice-cream, 400 of them wear glasses and like ice-cream, 250 of them are Democrats who wear glasses and like ice-cream. How many of them are not Democrats, do not wear glasses, and do not like ice-cream? How many of them are Democrats, who do not wear glasses, and do not like ice-cream? A box weighing 660 N is sliding across a cement floor and the coefficient of slidingfriction between the box and the floor is 0.15. What is the frictional force?09gO 4,400 N0 99 NO 99 kg PLEASE FIND THE COORDINATES!!! How is the Assembly of ancient Athens like the US Congress? O They both elect a president. O They both make and vote on laws. O They both are made up of elected members. O They both have the same number of members. 4. The following set of points belong to a specific function: {(-3,0)(-2,4), (-1,0 ), (0,-6),(1,-8), (2,0),(3,24)} Based on the set of points answers the following questions:a) What type of function does the set of points produce? Justify your answer.b) Write an equation for this function based on the set of points that have been given. A driver was operating his vehicle along a residential street at the posted speed limit when he saw a ball roll across the street. The driver did not slow down because the ball cleared his path before he reached it. A few seconds later, a child darted out into the street after the ball without looking for cars. The driver's car struck the child, and the child was injured. The child's parents brought an action on the child's behalf against the driver in a jurisdiction that follows traditional contributory negligence rules. At trial, the above facts were established. At the close of the evidence, the driver moved for a directed verdict. What should the court do Which expression is equivalent to [tex]\sqrt{100}[/tex]10105050 What is Bacteria?.. Which liquid would be the most difficult to raise or lower the temperature of?OA. One with a low densityOB. One with a high densityC. One with a low heat capacityOD. One with a high heat capacitySUSMIT The middle ear provides impedance matching between air and fluid by? Directions: Imagine that you are a journalist tasked with writing about the Great Migration of 1910 - 1930 and how it has brought about the Harlem Renaissance. In no less than 500 words, describe what was occurring in the U.S. South during this time and why so many African Americans decided to relocate to other parts of the country. Secondly, describe how the migration contributed to the Harlem Renaissance. Describe the Renaissance and the accomplishments of at least (2) African Americans during this time. Here are these directions showing what to include in your writing in list format:* Describe the Great Migration and why it is happening* Describe how the Migration is contributing to the Harlem Renaissance * Describe was the Harlem Renaissance is and write about the accomplishments of (2) or more African Americans who have contributed to the cultural movement How do u find the fraction result of this equation,Im confused what is the value of h(2)a) -2 b) -1c) 0d) 3 You have discovered the person giving you a ride home has been drinking. Use the GREAT decision making model to complete the R and E step. Think of at least 3 options and evaluate the pros and cons for each of your optionsOption1:list a pro and con for option 1Option 2:list a pro and con for option 2Option 3:list a pro and con for option 3 what is the formal charge of the central nitrogen atom in the best lewis structure dinitrogen sulfide